Last visit was: 25 Apr 2024, 22:06 It is currently 25 Apr 2024, 22:06

Close
GMAT Club Daily Prep
Thank you for using the timer - this advanced tool can estimate your performance and suggest more practice questions. We have subscribed you to Daily Prep Questions via email.

Customized
for You

we will pick new questions that match your level based on your Timer History

Track
Your Progress

every week, we’ll send you an estimated GMAT score based on your performance

Practice
Pays

we will pick new questions that match your level based on your Timer History
Not interested in getting valuable practice questions and articles delivered to your email? No problem, unsubscribe here.
Close
Request Expert Reply
Confirm Cancel
SORT BY:
Date
Math Expert
Joined: 02 Sep 2009
Posts: 92915
Own Kudos [?]: 619057 [23]
Given Kudos: 81595
Send PM
Most Helpful Reply
Math Expert
Joined: 02 Sep 2009
Posts: 92915
Own Kudos [?]: 619057 [8]
Given Kudos: 81595
Send PM
General Discussion
avatar
Intern
Intern
Joined: 29 Jul 2012
Posts: 1
Own Kudos [?]: 2 [2]
Given Kudos: 2
Send PM
Math Expert
Joined: 02 Sep 2009
Posts: 92915
Own Kudos [?]: 619057 [0]
Given Kudos: 81595
Send PM
Re: M27-12 [#permalink]
Expert Reply
eureka750 wrote:
The same applicable to (x+y)^2 ==> x^2 + y^2 + 2 x*y is greter than or equal to ZERO.
1 + 2 x*y is greter than or equal to ZERO.
xy greter than or equal -1/2...This can prove A is insufficient, right ???


You need to consider \((x-y)^2\geq{0}\) to get sufficiency.
avatar
Intern
Intern
Joined: 24 Feb 2015
Posts: 10
Own Kudos [?]: [0]
Given Kudos: 21
GMAT 1: 570 Q37 V31
Send PM
Re: M27-12 [#permalink]
Why did you start from : (x-y)^2>=0 ??
avatar
Intern
Intern
Joined: 09 Feb 2015
Posts: 2
Own Kudos [?]: 2 [0]
Given Kudos: 31
Send PM
Re: M27-12 [#permalink]
Bunuel wrote:
eureka750 wrote:
The same applicable to (x+y)^2 ==> x^2 + y^2 + 2 x*y is greter than or equal to ZERO.
1 + 2 x*y is greter than or equal to ZERO.
xy greter than or equal -1/2...This can prove A is insufficient, right ???


You need to consider \((x-y)^2\geq{0}\) to get sufficiency.



Please explain why do we need to consider only (x-y)^2 to get sufficiency, as also in (x+y)^2 we use x^2 + y^ 2=1 premise, hence its not in all cases that(A) results in single conclusion or sufficiency.
avatar
Intern
Intern
Joined: 09 May 2014
Posts: 3
Own Kudos [?]: 1 [0]
Given Kudos: 81
Send PM
Re: M27-12 [#permalink]
yeah even i have a same doubt why not (x+y)^2 as harshalnamdeo88...
Math Expert
Joined: 02 Sep 2009
Posts: 92915
Own Kudos [?]: 619057 [3]
Given Kudos: 81595
Send PM
M27-12 [#permalink]
2
Kudos
1
Bookmarks
Expert Reply
BCCLSN wrote:
Why did you start from : (x-y)^2>=0 ??

harshalnamdeo88 wrote:
Bunuel wrote:
eureka750 wrote:
The same applicable to (x+y)^2 ==> x^2 + y^2 + 2 x*y is greter than or equal to ZERO.
1 + 2 x*y is greter than or equal to ZERO.
xy greter than or equal -1/2...This can prove A is insufficient, right ???


You need to consider \((x-y)^2\geq{0}\) to get sufficiency.



Please explain why do we need to consider only (x-y)^2 to get sufficiency, as also in (x+y)^2 we use x^2 + y^ 2=1 premise, hence its not in all cases that(A) results in single conclusion or sufficiency.

shrutikasat wrote:
yeah even i have a same doubt why not (x+y)^2 as harshalnamdeo88...


The key is that if you look at \((x+y)^2 \ge 0\), you'll find \(xy\geq{-\frac{1}{2} }\). But this isn't really helpful. It's true that both \(xy\geq{-\frac{1}{2} }\) (from \((x+y)^2 \ge 0\)) and \(xy \le \frac{1}{2}\) (from \((x-y)^2 \ge 0\)) are correct. So, this means \(-\frac{1}{2} \leq{xy} \leq{\frac{1}{2} }\). But only the method shown in the original solution gets us the answer we want. The other way just gives us an inequality that doesn't help much.

Hope it's clear.
Intern
Intern
Joined: 06 Apr 2017
Posts: 21
Own Kudos [?]: 102 [10]
Given Kudos: 38
Location: United States (OR)
Concentration: Finance, Leadership
GMAT 1: 730 Q49 V40
GPA: 3.98
WE:Corporate Finance (Health Care)
Send PM
Re: M27-12 [#permalink]
9
Kudos
1
Bookmarks
This is a hard question, in particular because knowing to use the square of a difference is a tricky detail.

You can also use the fact that the maximum product of \(ab\) given that \(a+b=c\) is the set of numbers where \(a\) and \(b\) are nearest each other and still sum to \(c\).

For example, if \(a+b=20\), the maximum product of \(ab=10*10=100\)

In this case the maximum product of \(ab\) where \(a+b=1\) is when \(a=b=\frac{1}{2}\)

In this case \(x^2=a\) and \(y^2=b\), so the the maximum product of \(xy\) is \(\sqrt{\frac{1}{2}}*\sqrt{\frac{1}{2}}=\frac{1}{2}\)

Answer A
Volunteer Expert
Joined: 16 May 2019
Posts: 3512
Own Kudos [?]: 6860 [1]
Given Kudos: 500
Re: M27-12 [#permalink]
1
Kudos
Expert Reply
Bunuel wrote:
Is \(xy \le \frac{1}{2}\)?

(1) \(x^2+y^2=1\)

(2) \(x^2-y^2=0\)

Hello, everyone. If it helps, I thought I would add a graphical solution. Notice that the equation in Statement (1) conforms to that for a circle:

\((x-h)^2+(y-k)^2=r^2\)

in which (h, k) is the center of the circle and r is the radius. This circle would fit the equation thus:

\((x - 0)^2 + (y-0)^2=1^2\)

\(x^2 + y^2=1\)

This is the very unit circle that many trigonometry and pre-calculus students (at least, in the US) are taught, and recognizing it here can save a lot of time and effort.

Attachment:
Screen Shot 2022-04-18 at 14.26.33.png
Screen Shot 2022-04-18 at 14.26.33.png [ 29.03 KiB | Viewed 1151 times ]

Where it touches the circle, the green segment represents the maximum value for xy, since, in quadrants I and III, the two unknowns will have the same sign. The maximum value for the product will be found at 45 (or 225) degrees, equidistant from the x- and y- axes. (You can use a 45-45-90 special right triangle if you like. You know that in the corresponding side ratios x, x, and x√2, respectively, 1, the radius of the circle, is equivalent to x√2. This leads to a leg length of 1/√2 or, when the denominator is rationalized, √2/2.)

Thus, the maximum value of xy is

\(\frac{1}{\sqrt{2}}*\frac{1}{\sqrt{2}} = \frac{1}{2}\)

Since 1/2 is equal to 1/2, but we know the product cannot be any greater, Statement (1) is SUFFICIENT.

Statement (2) should take little more than a glance to see that the equation will prove insufficient on its own. Just as Bunuel has done above, start by rearranging.

\(x^2-y^2=0\)

\(x^2 = y^2\)

\(|x| = |y|\)

On its own, the statement tells us nothing about the nature of xy, relative to 1/2.

The answer must be (A).

Although knowledge of the circle equation and the unit circle is not required for the GMAT™, it can come in handy in unexpected ways, sometimes on tougher questions such as this one.

Good luck with your studies.

- Andrew
Math Expert
Joined: 02 Sep 2009
Posts: 92915
Own Kudos [?]: 619057 [0]
Given Kudos: 81595
Send PM
Re: M27-12 [#permalink]
Expert Reply
I have edited the question and the solution by adding more details to enhance its clarity. I hope it is now easier to understand.
GMAT Club Bot
Re: M27-12 [#permalink]
Moderator:
Math Expert
92915 posts

Powered by phpBB © phpBB Group | Emoji artwork provided by EmojiOne